Cauchy sequence with a convergent subsequence

Click For Summary
In a metric space, a Cauchy sequence (xn) that has a convergent subsequence (xn_k) converging to a point a implies that the entire sequence (xn) also converges to a. It is sufficient to demonstrate that just one subsequence converges to a, rather than requiring all subsequences to do so. The proof involves showing that for any ε > 0, the terms of the Cauchy sequence can be made arbitrarily close to a by leveraging the properties of both the Cauchy condition and the convergence of the subsequence. Specifically, by choosing appropriate indices, one can establish that the distance between terms in the sequence and the limit a can be made less than any arbitrary ε. The discussion emphasizes the importance of understanding the relationship between Cauchy sequences and convergence in metric spaces.
kingwinner
Messages
1,266
Reaction score
0

Homework Statement


Theorem: In a metric space X, if (xn) is a Cauchy sequence with a subsequence (xn_k) such that xn_k -> a, then xn->a.

Homework Equations


N/A

The Attempt at a Solution


1) According to this theorem, if we can show that ONE subsequence of xn converges to a, is that enough? Or do we need to show that EVERY subseuqence of xn converges to a in order to claim that xn->a?

2) How can we prove the theorem?
By definition, xn is Cauchy iff for all ε>0, there exists N s.t. if n,m≥N, then d(an,am)<ε.
By definition, xn->a iff for all ε>0, there exists M s.t. if n≥M, then d(xn,a)<ε.
I know the definitions, but I don't see how to link these all together to prove the theorem...

Any help is greatly appreciated!
 
Physics news on Phys.org
kingwinner said:
1) According to this theorem, if we can show that ONE subsequence of xn converges to a, is that enough? Or do we need to show that EVERY subseuqence of xn converges to a in order to claim that xn->a?

You can only assume it for one subsequence. Otherwise you could just take the subsequence n_k=k and be done.

The general idea of this proof is that for a_nk to converge to a means that for large enough N for all k>N a_nk is close to a. Now the issue here is that a_nk doesn't include all elements of a_n, but to say that a_n is Cauchy means precisely that for large enough indices the elements are close to each other.

To actually perform the proof let us choose some \epsilon &gt; 0. Note that since a_{n_k} \to a there exists some K>0 such that for all k>K we have d(a_{n_k},a) &lt; \epsilon. Now since (a_n) is Cauchy there exists some N>0 such that if n,m>N, then d(a_n,a_m) &lt; \epsilon. This is the necessary setup.

There exists some k' such that n_{k&#039;} &gt; N and k&#039; &gt; K. By the Cauchy condition if n&gt;N, then,
d(a_{n_{k&#039;}},a_n) &lt; \epsilon
Similarily by the convergence we have,
d(a_{n_{k&#039;}},a}) &lt; \epsilon
Add these and use the triangle inequality to get,
d(a_n,a) &lt; 2\epsilon
Since \epsilon was arbitrary this shows that (a_n) converges to a.
 
rasmhop said:
You can only assume it for one subsequence. Otherwise you could just take the subsequence n_k=k and be done.

The general idea of this proof is that for a_nk to converge to a means that for large enough N for all k>N a_nk is close to a. Now the issue here is that a_nk doesn't include all elements of a_n, but to say that a_n is Cauchy means precisely that for large enough indices the elements are close to each other.

To actually perform the proof let us choose some \epsilon &gt; 0. Note that since a_{n_k} \to a there exists some K>0 such that for all k>K we have d(a_{n_k},a) &lt; \epsilon. Now since (a_n) is Cauchy there exists some N>0 such that if n,m>N, then d(a_n,a_m) &lt; \epsilon. This is the necessary setup.

There exists some k' such that n_{k&#039;} &gt; N and k&#039; &gt; K. By the Cauchy condition if n&gt;N, then,
d(a_{n_{k&#039;}},a_n) &lt; \epsilon
Similarily by the convergence we have,
d(a_{n_{k&#039;}},a}) &lt; \epsilon
Add these and use the triangle inequality to get,
d(a_n,a) &lt; 2\epsilon
Since \epsilon was arbitrary this shows that (a_n) converges to a.
There exists some k' such that n_{k&#039;} &gt; N and k&#039; &gt; K <---can you please explain this part?

Thank you!
 
kingwinner said:
There exists some k' such that n_{k&#039;} &gt; N and k&#039; &gt; K <---can you please explain this part?

Thank you!

Well 1 \leq n_1 &lt; n_2 &lt; \cdots so n_i \geq i (n_1 must be at least 1, n_2 must be at least one more, n_3 must be at least one more than n_2, etc.). Thus n_{N+1} &gt; N so simply let k&#039; = \max(N+1,K+1), then
n_{k&#039;} \geq n_{N+1} &gt; N \qquad k&#039; \geq K+1 &gt; K
Basically what this says is the sequence (n_k) will eventually become greater than N (it's unbounded) and there exists an integer larger than K.
 
Question: A clock's minute hand has length 4 and its hour hand has length 3. What is the distance between the tips at the moment when it is increasing most rapidly?(Putnam Exam Question) Answer: Making assumption that both the hands moves at constant angular velocities, the answer is ## \sqrt{7} .## But don't you think this assumption is somewhat doubtful and wrong?

Similar threads

  • · Replies 1 ·
Replies
1
Views
2K
  • · Replies 1 ·
Replies
1
Views
2K
Replies
1
Views
5K
  • · Replies 2 ·
Replies
2
Views
3K
  • · Replies 5 ·
Replies
5
Views
2K
  • · Replies 5 ·
Replies
5
Views
2K
Replies
2
Views
3K
Replies
11
Views
3K
  • · Replies 8 ·
Replies
8
Views
3K
  • · Replies 5 ·
Replies
5
Views
2K